Find all subgroups of the octic group

  • Thread starter Shackleford
  • Start date
  • Tags
    Group
In summary, the conversation involves discussing the work done so far on finding all normal subgroups of the octic group. The participants also discuss the elements and subgroups of the group, including determining the order and normality of certain elements. They also mention the process of finding all subgroups of the octic group and provide tips for making the process more efficient.
  • #1
Shackleford
1,656
2
Here is my work thus far, and I included any pertinent notes.

http://i111.photobucket.com/albums/n149/camarolt4z28/IMG_20110802_182239.jpg?t=1312327545
 
Last edited by a moderator:
Physics news on Phys.org
  • #2
Looks good so far, shackleford! :smile:


I'll also assume that the "octic group" is just the dihedral group of order 8. I'm not familiar with the name octic group...
 
  • #3
micromass said:
Looks good so far, shackleford! :smile:


I'll also assume that the "octic group" is just the dihedral group of order 8. I'm not familiar with the name octic group...

It's the operations on the square numbered

4 3
1 2.

I'm missing an element from the subgroup of order 4.
 
  • #4
Shackleford said:
It's the operations on the square numbered

4 3
1 2.

Indeed, that would be the dihedral group. :smile:

I'm missing an element from the subgroup of order 4.

Well, what happens if you do (1 2 3 4)(1 2 3 4)?? Which familiar element do you get?
 
  • #5
micromass said:
Indeed, that would be the dihedral group. :smile:
Well, what happens if you do (1 2 3 4)(1 2 3 4)?? Which familiar element do you get?

Well, I figured I could simply do another permutation different than alpha and alpha-squared.

(1,3,2,4)
 
  • #6
micromass said:
Indeed, that would be the dihedral group. :smile:
Well, what happens if you do (1 2 3 4)(1 2 3 4)?? Which familiar element do you get?

(1 2 3 4)(1 2 3 4) = (1,3)(2,4) = α2

I already have (1,3)(2,4) in the subgroup of order 2.
 
  • #7
Shackleford said:
(1 2 3 4)(1 2 3 4) = (1,3)(2,4) = α2

I already have (1,3)(2,4) in the subgroup of order 2.

Your elements can occur in multiple subgroups. That's not forbidden...
 
  • #8
micromass said:
Your elements can occur in multiple subgroups. That's not forbidden...

I understand that, but the order is not four. The order of the permutation is the LCM of the order of the individual cycles.
 
  • #9
Also, the next problem is Find all normal subgroups of the octic group.

Wouldn't that be all the subgroups since they each have the identity in them? The left and right cosets would be equal since the identity captures all of the elements in the octic group.
 
  • #10
Shackleford said:
I understand that, but the order is not four. The order of the permutation is the LCM of the order of the individual cycles.

Yes, the order of a2 is 2. But why would that prevent {1,a,a2,a3} to form a subgroup??
 
  • #11
Shackleford said:
Also, the next problem is Find all normal subgroups of the octic group.

Wouldn't that be all the subgroups since they each have the identity in them? The left and right cosets would be equal since the identity captures all of the elements in the octic group.

What does having the identity has to do with normality??

Calculate the left and right cosets of [itex]\beta[/itex] for example. You'll see that the left and right cosets will not be equal...
 
  • #12
micromass said:
Yes, the order of a2 is 2. But why would that prevent {1,a,a2,a3} to form a subgroup??

Oh, you're right. The order of that element divides 4. In general then, the subgroup is not unique then because I could stick another second-order element in its place.
 
  • #13
micromass said:
What does having the identity has to do with normality??

Calculate the left and right cosets of [itex]\beta[/itex] for example. You'll see that the left and right cosets will not be equal...

Look at my subgroups. Each of the subgroups have the identity.

Normality is set equality.

xH = Hx for all x in G.

H = G1,...,G8 = G.
 
  • #14
Shackleford said:
Oh, you're right. The order of that element divides 4. In general then, the subgroup is not unique then because I could stick another second-order element in its place.

No, you couldn't :biggrin:

Having a in the subgroup will force a2 in the subgroup, by definition.
 
  • #15
Shackleford said:
Look at my subgroups. Each of the subgroups have the identity.

Yes, all subgroups of every group will contain the identity. This has nothing to do with normality.
 
  • #16
micromass said:
No, you couldn't :biggrin:

Having a in the subgroup will force a2 in the subgroup, by definition.

Oh. The subgroup has to be closed. That's why alpha-squared has to be in there.
 
  • #17
micromass said:
Yes, all subgroups of every group will contain the identity. This has nothing to do with normality.

Duh. Sorry. I had a brain fart there. I need to look at the product of each subgroup with each element in G to determine if the subgroup is normal in G.

I have the handy group table in the back of the book.
 
  • #18
Okay. I found that the only two normal subgroups of the octic group are, according to my notation, G1 = {e} and G2 = {e, alpha2}. G is not normal in G.
 
  • #19
Shackleford said:
Okay. I found that the only two normal subgroups of the octic group are, according to my notation, G1 = {e} and G2 = {e, alpha2}. G is not normal in G.

These aren't all the normal subgroups yet, you're missing one. (Hint: subgroups of index 2 are always normal)

By the way, I'm also not convinced that you actually found all the subgroups either...
 
  • #20
micromass said:
These aren't all the normal subgroups yet, you're missing one. (Hint: subgroups of index 2 are always normal)

By the way, I'm also not convinced that you actually found all the subgroups either...

Then I have no idea how to find them all. How do I find all subgroups of the octic group?
 
Last edited:
  • #21
Shackleford said:
Then I have no idea how to find them all. How do I find all subgroups of the octic group?

Well, first you take an arbitrary element x and see what the subgroup generated by x is. This will give you all cyclic subgroups of the octic group. This is what you have done.

But you're not done yet. There-after, you will have to take two arbitrary elements x and y, and you'll have to see what kind of group that those two elements generate.

Then you'll have to take 3 elements and see what they generate. And so on...

This sounds like a lot of work, but it isn't. If you're smart, then you can cut a lot of work.

For example, let's say that you are in STEP 2 and you see what group is generated by 2 elements. Obviously, you don't need to check it for a and a2, since these two elements will lie in the same cyclic subgroup. And obviously, the group generated by a and delta is the same as the group generated by a3 and delta.
 
  • #22
It also looks like G7 is a normal subgroup, too.
 
  • #23
Okay. I generated the subgroups based on the distinct powers of the elements of G. The only nontrivial generated subgroups are

<α> = {α, α2, α3, e}
3> = {α, α2, α3, e}

I'm not smart. I'm not following your shortcut.

Do I multiply α, α2, α3 by each of the other elements? I see they did that for A = {alpha, beta}.

If I do alpha with beta, gamma, theta, and delta I get {gamma, delta, beta, and theta}. It looks like I would get similar results with alpha squared and cubed.
 
Last edited:
  • #24
Okay, I found that someone had already worked out this problem in response to a question on a different website.

If you adjoin any element with <a> or <a3> you get G. How do you quickly see this? Just by looking at the table?

Adjoining the other elements with <a2> gives you two distinct subgroups of order 4. I suppose I'm missing another subgroup of order 4. Well, now, I have these two new subgroups, the identity, but it would appear I'm missing one.

http://i111.photobucket.com/albums/n149/camarolt4z28/20110804161007509.jpg?t=1312492825
 
Last edited by a moderator:
  • #25
Shackleford said:
Okay, I found that someone had already worked out this problem in response to a question on a different website.

If you adjoin any element with <a> or <a3> you get G. How do you quickly see this? Just by looking at the table?

<a> has order 4, adjoining an element to <a> gives you a subgroup with order at least 5. But by Lagrange's theorem, the order must divide 8. So adjoining an element to <a> gives you a group of order 8: the entire group.

[/QUOTE]
Adjoining the other elements with <a2> gives you two distinct subgroups of order 4. I suppose I'm missing another subgroup of order 4. Well, now, I have these two new subgroups, the identity, but it would appear I'm missing one.

http://i111.photobucket.com/albums/n149/camarolt4z28/20110804161007509.jpg?t=1312492825 [/QUOTE]

Why do you get the impression that you're missing one?
 
Last edited by a moderator:
  • #26
micromass said:
<a> has order 4, adjoining an element to <a> gives you a subgroup with order at least 5. But by Lagrange's theorem, the order must divide 8. So adjoining an element to <a> gives you a group of order 8: the entire group.
Adjoining the other elements with <a2> gives you two distinct subgroups of order 4. I suppose I'm missing another subgroup of order 4. Well, now, I have these two new subgroups, the identity, but it would appear I'm missing one.

http://i111.photobucket.com/albums/n149/camarolt4z28/20110804161007509.jpg?t=1312492825 [/QUOTE]

Why do you get the impression that you're missing one?[/QUOTE]

Well, you told me I was missing a couple of subgroups. I found two new subgroups of order 4. I'm mistaken. I need to check that the elements of this subgroup divide 4. Let me check.
 
Last edited by a moderator:
  • #27
Shackleford said:
Adjoining the other elements with <a2> gives you two distinct subgroups of order 4. I suppose I'm missing another subgroup of order 4. Well, now, I have these two new subgroups, the identity, but it would appear I'm missing one.

http://i111.photobucket.com/albums/n149/camarolt4z28/20110804161007509.jpg?t=1312492825

Why do you get the impression that you're missing one?

Well, you told me I was missing a couple of subgroups. I found two new subgroups of order 4. I'm mistaken. I need to check that the elements of this subgroup divide 4. Let me check.

Yes, the subgroups you found are ok. In total, there are 10 subgroups! So you found them all!
 
Last edited by a moderator:
  • #28
micromass said:
Yes, the subgroups you found are ok. In total, there are 10 subgroups! So you found them all!

How do I know that's all? I'm trying to figure out the best approach.

In finding the distinct subgroups of a group, I know to look at the sets generated by the powers of each of the elements. I also need to look at the other possible combinations.

The set generated by powers of alpha yields a subgroup of order 4. It seems that the order of the subgroups in looking at the other possible combinations is important. Any other element adjoined with the set generated by alpha creates a subgroup that must jump up to 8 in order to divide 8. The other subgroups have order 2, so adjoining an additional element with each of them can generate a set that jumps to 4 or 8. How did we know to adjoin the other elements with alpha-squared? Is it because it's commutative?
 
  • #29
Shackleford said:
How do I know that's all? I'm trying to figure out the best approach.

In finding the distinct subgroups of a group, I know to look at the sets generated by the powers of each of the elements. I also need to look at the other possible combinations.

The set generated by powers of alpha yields a subgroup of order 4. It seems that the order of the subgroups in looking at the other possible combinations is important. Any other element adjoined with the set generated by alpha creates a subgroup that must jump up to 8 in order to divide 8. The other subgroups have order 2, so adjoining an additional element with each of them can generate a set that jumps to 4 or 8. How did we know to adjoin the other elements with alpha-squared? Is it because it's commutative?

We didn't know that we had to adjoing a2. We have to adjoin every possible element to our subgroups and show that you get nothing else.

So you have found all subgroups, but you might still want to show that you found them all.
 
  • #30
micromass said:
We didn't know that we had to adjoing a2. We have to adjoin every possible element to our subgroups and show that you get nothing else.

So you have found all subgroups, but you might still want to show that you found them all.

Really? Adjoin every element with the cyclic subgroups? This is a ridiculous, tedious problem.
 
  • #31
Shackleford said:
Really? Adjoin every element with the cyclic subgroups? This is a ridiculous, tedious problem.

It can be done very economically! You need to do very few calculations!

Let me take two arbitrary example

Take [itex]\theta[/itex] and [itex]\gamma[/itex]. Adjoining these together would give me a subgroup of at least order 4 (indeed, e needs to be in the subgroup, so the subgroup has at least order 3. So by Lagrange, it has at least order 4). But [itex]\{\theta,\gamma,e,a^2\}[/itex] is such a subgroup.

Take a and [itex]\Delta[/itex]. In the subgroup generated by these would have to be

[tex]e,a,a^2,a^3,\Delta[/tex]

So the group has at least order 5. So by Lagrange, it has order 8.

These reasonings go very quickly. You'll end up with very few calculations!
 
  • #32
I still have to look at the various combinations of alpha squared, beta, gamma, delta, and theta. I can reason they should yield subgroups of order 4. I would have to still look at each of them to find the distinct subgroups. There has to be a shortcut to know which is the magic element.
 
  • #33
i think you are in my class. I emailed proof about this problem. I will copy and paste. If you are not in my class, then i hope this helps.

Me:#14) Find all subgroups of the octic group.

To understand this question, I am trying to understand example 5 which lists the subgroups of S3.

So, S3 has an order of 6 so all the subgroups will have the order of 1, 2, 3, or 6.

So, the subgroups with an order of 1 are only {(1)}. The subgroups with order 2 are {(1), (1, 2)}; {(1), (1,3)}, {(1), (2, 3)}. But why aren't other combinations of those element also subgroups ? Like, {(1), (1, 3), (2, 3)}? Doesn't this have an order of 2 as well? Continuing, the book lists the next subgroup as {(1), (1, 2, 3), (1, 3, 2)} which has order of 3. But, in the same vein as my question above, why are {(1), (1, 2, 3)} and {(1), (1, 3, 2)} not subgroups? Don't they have an order of three as well?

Proff:

Remember that a subgroup has to be closed with respect to multiplication. So if the subgroup contains (1 3 2) it must also contain its square which is (1 2 3)
If a subgroup contains (1 3 2) it must also contain ITS square which is (1 2 3) so you can't have a subgroup with one and not the other.

If a subgroup contains an element, it must also contain all powers of the element. That is why you can't have a subgroup with just (1) and (1 2 3)

If a subgroup contains 2 elements, it must also contain all possible products, so if a and b are in the set, so are a*a, a*b, b*a, b*b, a*b*a, a*b*b,...
With a small finite group there are only so many of these products that are actaually distinct.

Me:

I think i understand now.

So, the octic group has order of 8, so the subgroups have the order of 1, 2, 4, or 8.

The elements of the octic group by order are, e (order of 1), a^2, b, y, Delta, theta (all order of two), and a , a^3 (order of 4)

So H1 = {e} because it has an order of 1 and it is closed
H2 = {e, a^2} this has an order of 2 and it is closed bc e*e=e, e*a^2 =a^2*e=a^2, and a^2 *a^2=e
H3 = {e, b} same reason as above
H4 = {e, y} same reason
H5 = {e, Delta} same reason
H6= {e, theta) same reason

H7 = {e, a, a^2} this has order of 4, and a^2 was added so that the subgroup will be closed
H8 = {e, a^3, a^2} same reason

H9 = G

My method is to go element by element so long as it follows the order rule, and add the squares (or other elements) if needed.

Would you suppose this works?
Proff:
Almost,

remember if you have a, you also have a^2 and a^3 (and a^4, a^5,...

So, one subgroup is e, a, a^2, a^3
However, reading this post makes me think i have my subgroups incorrect. With my current subgroups, i got H1, H2, H9, H10 as normal
 
  • #34
Oh, i changed H9 to = {e, a, a^2, a^3} and H10 =G
 
  • #35
also, i am looking at the most recent pic that you posted and I don't understand how you got the two subsets on the right side of the page.
 

Similar threads

Replies
19
Views
3K
Replies
5
Views
1K
Replies
7
Views
2K
Replies
11
Views
2K
Replies
15
Views
3K
Replies
1
Views
983
Replies
1
Views
2K
Back
Top